Board logo

标题: gwd13-7 [打印本页]

作者: ytrrew    时间: 2005-1-18 19:54     标题: gwd13-7

GWD-13

Q7:

For a certain race, 3 teams were allowed to enter 3 members each. A team earned 6 – n points whenever one of its members finished in nth place, where 1 ≤ n ≤ 5. There were no ties, disqualifications, or withdrawals. If no team earned more than 6 points, what is the least possible score a team could have earned?

A. 0

B. 1

C. 2

D. 3

E. 4

Ans: D

Dont understand the question


作者: mikejia    时间: 2005-1-19 01:44

5+4+3+2+1=15. 某两个最多各6,剩下一个就是3了。

这道题问过若干次了,可以背出来了






欢迎光临 国际顶尖MBA申请交流平台--TOPWAY MBA (http://forum.topway.org/) Powered by Discuz! 7.2